Những câu hỏi liên quan
Đinh Gia Khánh
Xem chi tiết
Nguyễn Thùy Trang
Xem chi tiết
Minh Hiếu
Xem chi tiết
Nguyễn Việt Lâm
12 tháng 1 2022 lúc 0:02

1.

\(x^4+4y^4=x^4+4x^2y^2+y^4-4x^2y^2=\left(x^2+2y^2\right)^2-\left(2xy\right)^2\)

\(=\left(x^2-2xy+2y^2\right)\left(x^2+2xy+2y^2\right)\)

Do x, y nguyên dương nên số đã cho là SNT khi:

\(x^2-2xy+2y^2=1\Rightarrow\left(x-y\right)^2+y^2=1\)

\(y\in Z^+\Rightarrow y\ge1\Rightarrow\left(x-y\right)^2+y^2\ge1\)

Đẳng thức xảy ra khi và chỉ khi \(x=y=1\)

Thay vào kiểm tra thấy thỏa mãn

2. \(N=n^4+4^n\)

- Với n chẵn hiển nhiên N là hợp số

- Với \(n\) lẻ: \(\Rightarrow n=2k+1\)

\(N=n^4+4^n=n^4+4^{2k+1}=n^4+4.4^{2k}+4n^2.4^k-n^2.4^{k+1}\)

\(=\left(n^2+2.4^k\right)^2-\left(n.2^{k+1}\right)^2=\left(n^2+2.4^k-n.2^{k+1}\right)\left(n^2+2.4^k+n.2^{k+1}\right)\)

Mặt khác:

\(n^2+2.4^k-n.2^{k+1}\ge2\sqrt{2n^2.4^k}-n.2^{k+1}=2\sqrt{2}n.2^k-n.2^{k+1}\)

\(=n.2^{k+1}\left(\sqrt{2}-1\right)\ge2\left(\sqrt{2}-1\right)>1\)

\(\Rightarrow N\) là tích của 2 số dương lớn hơn 1

\(\Rightarrow\) N là hợp số

Bình luận (0)
Nguyễn Việt Lâm
12 tháng 1 2022 lúc 15:09

Bài 4 chắc không có cách "đại số" nào (tức là dựa vào lý luận chia hết tổng quát) để giải. Mình nghĩ vậy (có lẽ có, nhưng mình ko biết).

Chắc chỉ sáng lọc và loại trừ theo quy tắc kiểu: do đổi vị trí bất kì đều là SNT nên không thể chứa các chữ số chẵn và chữ số 5, như vậy số đó chỉ có thể chứa các chữ số 1,3,7,9

Nó cũng không thể chỉ chứa các chữ số  3 và 9 (sẽ chia hết cho 3)

Từ đó sàng lọc được các số: 113 (và các số đổi vị trí), 337 (và các số đổi vị trí)

Bình luận (9)
Minz Ank
Xem chi tiết
Nguyễn Việt Lâm
8 tháng 3 2023 lúc 20:48

Bài này yêu cầu tìm m; n nguyên hay m; n nguyên dương em nhỉ?

Bình luận (3)
Uchiha Sakura
Xem chi tiết
Tiến Nguyễn Minh
Xem chi tiết
Tiến Nguyễn Minh
Xem chi tiết
✰๖ۣۜŠɦαɗøω✰
27 tháng 3 2020 lúc 8:31

Bài 1 : 

Phương trình <=> 2x . x2 = ( 3y + 1 ) + 15

Vì \(\hept{\begin{cases}3y+1\equiv1\left(mod3\right)\\15\equiv0\left(mod3\right)\end{cases}\Rightarrow\left(3y+1\right)^2+15\equiv1\left(mod3\right)}\)

\(\Rightarrow2^x.x^2\equiv1\left(mod3\right)\Rightarrow x^2\equiv1\left(mod3\right)\)

( Vì số  chính phương chia 3 dư 0 hoặc 1 ) 

\(\Rightarrow2^x\equiv1\left(mod3\right)\Rightarrow x\equiv2k\left(k\inℕ\right)\)

Vậy \(2^{2k}.\left(2k\right)^2-\left(3y+1\right)^2=15\Leftrightarrow\left(2^k.2.k-3y-1\right).\left(2^k.2k+3y+1\right)=15\)

Vì y ,k \(\inℕ\)nên 2k . 2k + 3y + 1 > 2k .2k - 3y-1>0

Vậy ta có các trường hợp: 

\(+\hept{\begin{cases}2k.2k-3y-1=1\\2k.2k+3y+1=15\end{cases}\Leftrightarrow\hept{\begin{cases}2k.2k=8\\3y+1=7\end{cases}\Rightarrow}k\notinℕ\left(L\right)}\)

\(+,\hept{\begin{cases}2k.2k-3y-1=3\\2k.2k+3y+1=5\end{cases}\Leftrightarrow\hept{\begin{cases}2k.2k=4\\3y+1=1\end{cases}\Rightarrow}\hept{\begin{cases}k=1\\y=0\end{cases}\left(TM\right)}}\)

Vậy ( x ; y ) =( 2 ; 0 ) 

Bình luận (0)
 Khách vãng lai đã xóa
Phùng Gia Bảo
27 tháng 3 2020 lúc 9:14

Bài 3: 

Giả sử \(5^p-2^p=a^m\)    \(\left(a;m\inℕ,a,m\ge2\right)\)

Với \(p=2\Rightarrow a^m=21\left(l\right)\)

Với \(p=3\Rightarrow a^m=117\left(l\right)\)

Với \(p>3\)nên p lẻ, ta có

\(5^p-2^p=3\left(5^{p-1}+2.5^{p-2}+...+2^{p-1}\right)\Rightarrow5^p-2^p=3^k\left(1\right)\)    \(\left(k\inℕ,k\ge2\right)\)

Mà \(5\equiv2\left(mod3\right)\Rightarrow5^x.2^{p-1-x}\equiv2^{p-1}\left(mod3\right),x=\overline{1,p-1}\)

\(\Rightarrow5^{p-1}+2.5^{p-2}+...+2^{p-1}\equiv p.2^{p-1}\left(mod3\right)\)

Vì p và \(2^{p-1}\)không chia hết cho 3 nên \(5^{p-1}+2.5^{p-2}+...+2^{p-1}⋮̸3\)

Do đó: \(5^p-2^p\ne3^k\), mâu thuẫn với (1). Suy ra giả sử là điều vô lý

\(\rightarrowĐPCM\)

Bình luận (0)
 Khách vãng lai đã xóa
Lê Nhật Khôi
27 tháng 3 2020 lúc 10:53

Bài 4:

Ta đặt: \(S=6^m+2^n+2\)

TH1: n chẵn thì:

\(S=6^m+2^n+2=6^m+2\left(2^{n-1}+1\right)\)

Mà \(2^{n-1}+1⋮3\Rightarrow2\left(2^{n-1}+1\right)⋮6\Rightarrow S⋮6\)

Đồng thời S là scp

Cho nên: \(S=6^m+2\left(2^{n-1}\right)=\left(6k\right)^2\)

\(\Leftrightarrow6^m+6\left(2^{n-2}-2^{n-3}+...+2-1\right)=36k^2\)

Đặt: \(A\left(n\right)=2^{n-2}-2^{n-3}+...+2-1=2^{n-3}+...+1\)là số lẻ

Tiếp tục tương đương: \(6^{m-1}+A\left(n\right)=6k^2\)

Vì A(n) lẻ và 6k^2 là chẵn nên: \(6^{m-1}\)lẻ\(\Rightarrow m=1\)

Thế vào ban đầu: \(S=8+2^n=36k^2\)

Vì n=2x(do n chẵn) nên tiếp tục tương đương: \(8+\left(2^x\right)^2=36k^2\)

\(\Leftrightarrow8=\left(6k-2^x\right)\left(6k+2^x\right)\)

\(\Leftrightarrow2=\left(3k-2^{x-1}\right)\left(3k+2^{x-1}\right)\)

Vì \(3k+2^{x-1}>3k-2^{x-1}>0\)(lớn hơn 0 vì 2>0 và \(3k+2^{x-1}>0\))

Nên: \(\hept{\begin{cases}3k+2^{x-1}=2\\3k-2^{x-1}=1\end{cases}}\Leftrightarrow6k=3\Rightarrow k\notin Z\)(loại)

TH2: n là số lẻ

\(S=6^m+2^n+2=\left(2k\right)^2\)(do S chia hết cho 2 và S là scp)

\(\Leftrightarrow3\cdot6^{m-1}+2^{n-1}+1=2k^2\)là số chẵn

\(\Rightarrow3\cdot6^{m-1}+2^{n-1}\)là số lẻ

Chia tiếp thành 2TH nhỏ: 

TH2/1: \(3\cdot6^{m-1}\)lẻ và \(2^{n-1}\)chẵn với n là số lẻ

Ta thu đc: m=1 và thế vào ban đầu

\(S=2^n+8=\left(2k\right)^2\)(n lớn hơn hoặc bằng 3)

\(\Leftrightarrow2^{n-2}+2=k^2\)

Vì \(k^2⋮2\Rightarrow k⋮2\Rightarrow k^2=\left(2t\right)^2\)

Tiếp tục tương đương: \(2^{n-2}+2=4t^2\)

\(\Leftrightarrow2^{n-3}+1=2t^2\)

\(\Leftrightarrow2^{n-3}\)là số lẻ nên n=3

Vậy ta nhận đc: \(\left(m;n\right)=\left(1;3\right)\)

TH2/2: \(3\cdot6^{m-1}\)là số chẵn và \(2^{n-1}\)là số lẻ

Suy ra: n=1

Thế vào trên: \(6^m+4=4k^2\)

\(\Leftrightarrow6^m=\left(2k-2\right)\left(2k+2\right)\)

\(\Leftrightarrow\hept{\begin{cases}2k-2=6^q\\2k+2=6^p\end{cases}}\Rightarrow p+q=m\)

Và \(6^p-6^q=4\)

\(\Leftrightarrow6^q\left(6^{p-q}-1\right)=4\Leftrightarrow6^q\le4\Rightarrow q=1\)(do là tích 2 stn)

\(\Rightarrow k\notin Z\)

Vậy \(\left(m;n\right)=\left(1;3\right)\)

P/S: mk không kiểm lại nên có thể sai

Bình luận (0)
 Khách vãng lai đã xóa
Vân Lê
Xem chi tiết
NGUYỄN MINH HUY
Xem chi tiết
gãi hộ cái đít
12 tháng 3 2021 lúc 19:06

Ta có:

\(n^5+n^4-2n^3-2n^2+1=p^k\Leftrightarrow\left(n^2+n-1\right)\left(n^3-n-1\right)=p^k\)

Từ gt \(\Rightarrow n,k\ge2\)

Ta có:

\(\left\{{}\begin{matrix}n^3-n-1>1;n^2+n-1>1,\forall n\ge2\\\left(n^3-n-1\right)-\left(n^2+n-1\right)=\left(n+1\right)n\left(n-2\right)\ge0,\forall n\ge2\end{matrix}\right.\)

\(\Rightarrow\left\{{}\begin{matrix}n^3-n-1=p^r\\n^2+n-1=p^s\end{matrix}\right.\) trong đó \(\left\{{}\begin{matrix}r\ge s>0\\r+s=k\end{matrix}\right.\)

\(\Rightarrow n^3-n-1⋮n^2+n-1\)

\(\Rightarrow n^3-n-1-\left(n-1\right)\left(n^2+n-1\right)⋮n^2+n-1\)

\(\Rightarrow n-2⋮n^2+n-1\)       (1)

Mặt khác:

\(\left(n^2+n-1\right)-\left(n-2\right)=n^2+1>0,\forall n\)

\(\Rightarrow n^2+n-1>n-2\ge0,\forall n\ge2\) (2)

Từ (1) và (2) => n=2 => \(p^k=25\Rightarrow\left\{{}\begin{matrix}p=5\\k=2\end{matrix}\right.\)

Vậy bộ số (n,k,p)=(2,2,5)

Bình luận (0)
Trần Minh Hoàng
12 tháng 3 2021 lúc 18:34

\(...\Leftrightarrow\left(n^2+n-1\right)\left(n^3-n-1\right)=p^k\).

Do đó \(\left\{{}\begin{matrix}n^2+n-1=p^v\\n^3-n-1=p^u\end{matrix}\right.\left(v,u\in N;v+u=k\right)\).

+) Với n = 2 ta có \(p^k=25=5^2\Leftrightarrow p=5;k=2\) 

+) Với n > 2 ta có \(n^3-n-1>n^2+n-1\Rightarrow v>u\Rightarrow n^3-n-1⋮n^2+n-1\)

\(\Rightarrow\left(n^2+n-1\right)\left(n-1\right)+n-2⋮n^2+n-1\)

\(\Rightarrow n-2⋮n^2+n-1\)

\(\Rightarrow\left(n-2\right)\left(n+3\right)⋮n^2+n-1\)

\(\Rightarrow6⋮n^2+n-1\).

Không tồn tại n > 2 thoả mãn

Vậy...

 

 

 

Bình luận (0)